LSAT and Law School Admissions Forum

Get expert LSAT preparation and law school admissions advice from PowerScore Test Preparation.

 Administrator
PowerScore Staff
  • PowerScore Staff
  • Posts: 8927
  • Joined: Feb 02, 2011
|
#90650
Complete Question Explanation

The correct answer choice is E.

Answer choice (A):

Answer choice (B):

Answer choice (C):

Answer choice (D):

Answer choice (E): This is the correct answer choice.

This explanation is still in progress. Please post any questions below!
User avatar
 jailenea
  • Posts: 25
  • Joined: Aug 30, 2021
|
#101902
Can someone explain why E is correct and what the fastest way to get there is? R being 5th in place of L/N isn't yielding much information alone for me. I was testing every answer choice but had to guess (A) as I ran out of time.
 Luke Haqq
PowerScore Staff
  • PowerScore Staff
  • Posts: 712
  • Joined: Apr 26, 2012
|
#101935
Hi jailenea!

Answer choice (A) does initially look appealing. When one combines the 4th and 5th rules, it becomes clear that you can't have all three of I, R, and O at the same time. Thus it might seem that if R is in the 5th interview spot, then only one of I versus O will be chosen but not both. However, it's possible that neither of them is chosen, which is why (A) is ultimately incorrect.

Regarding (E), that answer choice states that T must be interviewed 4th. The 4th and 5th rules are again helpful here. In addition to suspending one of the rules, the question stem also tells that R is in spot 5. Because of that, we know that neither I nor O will be interviewed. This is because if you have I and R, then I must come after R, and if you have O and R, then O must come after R. In both cases, I and O must come after R, but there's no space since R is in the final spot.

With I and O eliminated, then we know the remaining variables are all used, since 5 of 7 will be interviewed. That is, we're left with G, L, N, R, and T. Based on the rules, there is only one possible ordering of these variables:

G, L, N, T, R
G must be first if G is used. L seems like it could be in spots 2 or 3, but we know that it must be in 2 because we need room for the NT block, since both N and T are used.

Get the most out of your LSAT Prep Plus subscription.

Analyze and track your performance with our Testing and Analytics Package.